Last visit was: 26 Apr 2024, 22:38 It is currently 26 Apr 2024, 22:38

Close
GMAT Club Daily Prep
Thank you for using the timer - this advanced tool can estimate your performance and suggest more practice questions. We have subscribed you to Daily Prep Questions via email.

Customized
for You

we will pick new questions that match your level based on your Timer History

Track
Your Progress

every week, we’ll send you an estimated GMAT score based on your performance

Practice
Pays

we will pick new questions that match your level based on your Timer History
Not interested in getting valuable practice questions and articles delivered to your email? No problem, unsubscribe here.
Close
Request Expert Reply
Confirm Cancel
SORT BY:
Date
Tags:
Show Tags
Hide Tags
User avatar
Manager
Manager
Joined: 09 Feb 2013
Posts: 104
Own Kudos [?]: 4042 [14]
Given Kudos: 17
Send PM
Most Helpful Reply
Math Expert
Joined: 02 Sep 2009
Posts: 92948
Own Kudos [?]: 619282 [10]
Given Kudos: 81609
Send PM
General Discussion
Verbal Forum Moderator
Joined: 10 Oct 2012
Posts: 485
Own Kudos [?]: 3093 [3]
Given Kudos: 141
Send PM
User avatar
Intern
Intern
Joined: 15 Feb 2013
Status:Currently Preparing the GMAT
Posts: 25
Own Kudos [?]: 58 [3]
Given Kudos: 11
Location: United States
GMAT 1: 550 Q47 V23
GPA: 3.7
WE:Analyst (Consulting)
Send PM
Re: If a and b are prime numbers, which of the following CANNO [#permalink]
3
Bookmarks
Prime numbers are a fundamental part in the GMAT, so having questions such as these are a real blessing. Thanks emmak (you've earned a Kudos from me).

I'll kick this off by stating which concepts are going to be useful in solving this problem :

1/ Addition laws : ODD + EVEN = ODD, ODD + ODD = EVEN
2/ All prime numbers except 2 are ODD !!!
3/ Prime number list up until 31
(Yeah, it's harsh, but you need to know your prime numbers, otherwise, you'll be missing out on a lot of shortcuts)

Now, let's consider the answer choices.

You'll notice that all the answer choices except D are odd. So there's got to be a 2 in the mix (if the question stem is correct) so :

A/ 19 = 2 + 17 (17 is prime, so cross off A) ;
B/ 25 = 2 + 23 (23 is prime, so cross off B) ;
C/ 33 = 2 + 31 (31 is prime, so cross off C) ;
D/ 46 = 23 + 23 (23 is prime, so cross off D);

This leaves us with answer choice E. And incidentally 51 IS a prime number. ;)

An alternative to this method is to use the list of prime numbers up to 31 and make combinations. Depending on how comfortable you are with numbers, this can be a time-saver or a time-consuming method. So use at your own risk.

Hope that helped :-D
Manager
Manager
Joined: 13 Oct 2013
Posts: 117
Own Kudos [?]: 200 [0]
Given Kudos: 530
Concentration: Strategy, Entrepreneurship
Send PM
If a and b are prime numbers, which of the following CANNOT be the sum [#permalink]
If a and b are prime numbers, which of the following CANNOT be the sum of a and b?

a) 19
b) 25
c) 33
d) 46
E) 53
SVP
SVP
Joined: 20 Mar 2014
Posts: 2362
Own Kudos [?]: 3626 [0]
Given Kudos: 816
Concentration: Finance, Strategy
GMAT 1: 750 Q49 V44
GPA: 3.7
WE:Engineering (Aerospace and Defense)
Send PM
Re: If a and b are prime numbers, which of the following CANNOT be the sum [#permalink]
sunita123 wrote:
If a and b are prime numbers, which of the following CANNOT be the sum of a and b?

a) 19
b) 25
c) 33
d) 46
E) 53


Please search for a question before you post. The same question has been discussed at if-a-and-b-are-prime-numbers-which-of-the-following-canno-149049.html?fl=similar
Math Expert
Joined: 02 Sep 2009
Posts: 92948
Own Kudos [?]: 619282 [0]
Given Kudos: 81609
Send PM
Re: If a and b are prime numbers, which of the following CANNO [#permalink]
Expert Reply
sunita123 wrote:
If a and b are prime numbers, which of the following CANNOT be the sum of a and b?

a) 19
b) 25
c) 33
d) 46
E) 53


Merging topics.

Please refer to the discussion above.
Manager
Manager
Joined: 24 Sep 2018
Posts: 107
Own Kudos [?]: 179 [0]
Given Kudos: 14
Send PM
Re: If a and b are prime numbers, which of the following CANNO [#permalink]
Quote:
In order for the sum of two integers to be odd, one of the numbers must be even. The only even prime number is 2. Therefore, if the sum of two prime numbers is odd, one of the numbers must be 2.


(A) 19 (odd) = 2 + 17

(B) 25 (odd) = 2 + 23

(C) 33 (odd) = 2 + 31

(D) 46 (even) = 3 + 43

(E) 53 (odd) = 2 + 51 (51 is not prime)

51 is not prime, so the correct answer is 53, answer choice E.

Quote:
Important: 51 is often used as a trap answer on prime number questions. 51 is a multiple of 3, and therefore not prime. You can check this using the divisibility rule for 3, as the sum of the digits (5+1 = 6) is a multiple of 3.
Intern
Intern
Joined: 05 Sep 2021
Posts: 1
Own Kudos [?]: 0 [0]
Given Kudos: 4
Send PM
Re: If a and b are prime numbers, which of the following CANNO [#permalink]
emmak wrote:
If a and b are prime numbers, which of the following CANNOT be the sum of a and b?

A. 19
B. 25
C. 33
D. 46
E. 53


Eliminate the answer choices by proving the possibility that an answer choice is a summation of prime numbers as follows:

A) 19 - 2 = 17 - Possible sum since 2 and 17 are prime
B) 25 - 2 = 23 - Possible (23 is a prime number)
C) 33 - 2 = 31 - Possible ( 31 is a prime number)
D) 46 -2 = 44 - 44 not a prime number. subtracting 3, 46 - 3 = 43, 43 and 3 are prime numbers
E) 53 - correct choice
GMAT Club Bot
Re: If a and b are prime numbers, which of the following CANNO [#permalink]
Moderators:
Math Expert
92948 posts
Senior Moderator - Masters Forum
3137 posts

Powered by phpBB © phpBB Group | Emoji artwork provided by EmojiOne